Stirling公式的几种经典估计

Stirling公式

对于$\Gamma$函数,我们有一个经典的极限式(证明请见ProofWiki)。

利用这个式子,我们能立刻计算出一些比较难算的极限。注意到这个公式如果写成自然数的形式,有

所以我们能立刻计算出这个极限:

但是Stirling公式不仅仅如此。这篇博客里我们会见到几个比较经典的估计。

原数列的取值区间

这一节我们会看到的结论是

如果在计算器里算一下右边的数,会发现,$\phi_n=\frac{n!}{(n/e)^n\sqrt{2\pi n}}$一直在$1$附近。

对于$m=1,2,3,\dots$,在$y=\ln(x)$下方定义“折线函数”:

其中$m \leq x \leq m+1$。在上方定义另一个“折线函数”:

其中$m-1/2 \leq x < m+1/2$。如果画出$f$,$\ln{x}$,$g$的图像,会发现,$f$和$g$是对$\ln{x}$的拟合。且在$x \geq 1$时,我们有

所以计算定积分的时候就有

但是$f$和$g$的关系并不是那么简单。计算$f$的积分,我们发现

而对于$g$,我们又有

这就说明

总结上面几个不等式,我们得到,对$n>1$:

不等式各项都减去$\int_1^n \ln x dx$,我们又有

由Stirling公式我们知道,

而数列$x_n=-\frac{1}{8n}+\ln(n!)-(\frac{1}{2}+n)\ln{n}+n$是单调递增的,由上式可知收敛到$\ln\sqrt{2\pi}$。在不等式左边,我们取上确界$\ln\sqrt{2\pi}$。在不等式右边,我们取下确界$x_1+\frac{1}{8}=1$。这就让我们得到了

这也就导致

这对所有$n =1,2,3,\dots$都成立。

平移$\Gamma$函数

对于任意$c \in \mathbb{R}$,我们有

这可以看成,把$\Gamma(x)$向左平移$c$后,在$x$足够大时,其值和$x^c\Gamma(x)$接近。这个等式的证明也是比较简单的,虽然计算比较繁琐,只需要利用Stirling公式。

现在这三个因式的极限就很好计算了。显然我们有

以及

最后,

故原极限为$1$。计算过程也非常精彩。注意到如果把$x$和$c$换成正整数$n$和整数$k$,我们又有

估计定积分

结合Bernoulli不等式我们有

接下来我们会给出一个比较精细的估计。实际上,

根据$B(x,y)$函数的定义,

令$t=u^2,我们得到

代入$x=\frac{1}{2}$和$y=n+1$,我们就和所想要的结果很近了:

注意到,利用$B$函数的第二个表达式,我们是可以计算出$\Gamma(\frac{1}{2})$的。实际上,

从而$\Gamma(\frac{1}{2})=\sqrt{\pi}$。对于$B(\frac{1}{2},n+1)$,我们可以用到上面的平移公式了:

从而

额外内容

最后我们证明一个和Stirling公式没有关系的等式

根据古典代数学基本定理,我们立刻有

注意到另一方面

$x=1$时,我们有

此即

考虑到欧拉反射公式,对于$1 \leq k \leq n-1$,我们有

如果$n$为奇数,那么根据上面的结果,我们能得到

这时我们只用到了一半数量的$k$。要用上另一半的$k$,我们只需要把$k$和$n-k$交换顺序,从而得到了

即为所得。如果$n$为偶数,只需要把$1/2$这一项单独拿出来分两段计算即可。


2020.11.9更新

我们给出两个看上去很难计算的极限式。

如果用Stirling公式直接替换$n!$,这个极限的结果是显然的。

所以只需要求$(1+\frac{1}{n})^{n^2}e^{-n}$的极限即可。但是可千万别想当然地认为这个极限是$1$。如果我们利用Taylor展开,能得到

所以原极限为$\sqrt\frac{2\pi}{e}$

注意$n$项的分子相乘,有$\exp(n-1-\frac{1}{2}-\cdots-\frac{1}{n})$,而调和级数是发散的,我们想得到收敛,自然就要想到Euler常数$\gamma=\lim_{n\to\infty}\left(1+\frac{1}{2}+\cdots+\frac{1}{n}-\ln{n}\right)$。我们似乎也没有办法直接化简分母,我们知道$(1+1/k)^k$的极限是$e$,但是这里似乎用不上。所以不如先把分母展开化简一下。

所以原极限可以写成

这时候就可以直接使用Stirling公式了。

而$\lim_{n\to\infty}\left(1+\frac{1}{n}\right)^{-n}=e^{-1}$,$\lim_{n\to\infty}e^{\ln{n}-1-\frac{1}{2}-\frac{1}{3}-\cdots-\frac{1}{n}}=e^{-\gamma}$,我们得到原极限为$\frac{\sqrt{2\pi}}{e^{1+\gamma}}$

Stirling公式的几种经典估计

https://desvl.xyz/2020/10/18/stirling-approximation/

Author

Desvl

Posted on

2020-10-18

Updated on

2023-07-08

Licensed under

Comments